行列式入门与矩阵树定理完整证明

前置技能

行列式

定义

对于一个 n × n n \times n 的矩阵 A A ,记它第 i i 行第 j j 列的元素为 a i , j a_{i, j} ,以及一个 1 n 1 \sim n 的排列 p p ,记 λ A ( p ) = ( 1 ) τ ( p ) i = 1 n a i , p i \lambda_A(p) = (-1) ^ {\tau(p)} \prod_{i = 1}^{n} a_{i, p_i} (其中 τ ( p ) \tau(p) p p 的逆序对个数)则 A = det A = p λ A ( p ) = p ( ( 1 ) τ ( p ) i = 1 n a i , p i ) |A| = \det A = \sum_{p} \lambda_A(p) = \sum_{p} \left( (-1) ^ {\tau(p)} \prod_{i = 1}^{n} a_{i, p_i} \right) (其中 A |A| det A \det A 都是 A A 的行列式的意思,后文有时为了便于区分行列式和绝对值会使用 det A \det A ,其余大部分时候使用 A |A| )显然行列式运算结果是数量。

简单理解一下发现这样的计算方法的时间复杂度是 O ( n log n n ! ) O(n \cdot \log n \cdot n!) 因此这个定义没什么用

性质

除了少数几个,名字基本都是乱编的

拉普拉斯展开

对于一个 n × n n \times n 的矩阵 A A 和任意一个 1 i n 1 \leq i \leq n ,有 A = j = 1 n ( 1 ) i + j a i , j A i , j |A| = \sum_{j = 1}^{n} (-1)^{i + j} a_{i, j} |A_{i, j}| 其中 A i , j A_{i, j} 指矩阵 A A 删去第 i i 行和第 j j 列的所有元素后形成的一个 ( n 1 ) × ( n 1 ) (n - 1) \times (n - 1) 的矩阵。这个计算式称为对第 i i 行进行拉普拉斯展开。

证明:
根据定义每个 λ A ( p ) \lambda_A(p) 一定有一个因子 x x 满足 x { a i , 1 , a i , 2 , , a i , n } x \in \{a_{i, 1}, a_{i, 2}, \cdots, a_{i, n}\} ,枚举 j j 得到这个因子 a i , j a_{i, j} 和满足 p i = j p_i = j 的排列 p p ,然后将这个 p i p_i 删除,并把 p p 中所有大于 j j 的元素减一得到 q q q q 1 n 1 1 \sim n-1 的排列),于是在 n , i , j n, i, j 确定时有了一个 p p q q 之间的一一对应的函数关系。

例如,当 n = 3 , i = 1 , j = 2 n = 3, i = 1, j = 2 时,满足 p 1 = 2 p_1 = 2 p p { 2 , 1 , 3 } , { 2 , 3 , 1 } \{2, 1, 3\}, \{2, 3, 1\} ,将 2 2 删除,得到 { 1 , 3 } , { 3 , 1 } \{1, 3\}, \{3, 1\} ,然后将 3 3 减一,得到 q q { 1 , 2 } \{1, 2\} { 2 , 1 } \{2, 1\} ;根据 n = 3 , i = 1 , j = 2 n = 3, i = 1, j = 2 ,亦可将 q q 还原为 p p

ξ ( p ) = q \xi(p) = q ,那么 p λ A ( p ) [ j = p i ] a i , j = p λ A i , j ( ξ ( p ) ) [ j = p i ] \left| \frac{\sum_{p} \lambda_A(p) \cdot [j = p_i]}{a_{i, j}} \right| = \left|\sum_{p} \lambda_{A_{i, j}}(\xi(p)) \cdot [j = p_i] \right| 右边变为直接枚举 ξ ( p ) \xi(p) ,即 p λ A ( p ) [ j = p i ] a i , j = q λ A i , j ( q ) = det A i , j \left| \frac{\sum_{p} \lambda_A(p) \cdot [j = p_i]}{a_{i, j}} \right| = \left|\sum_{q} \lambda_{A_{i, j}}(q) \right| = |\det A_{i, j}| p λ A ( p ) [ j = p i ] = a i , j det A i , j \left| \sum_{p} \lambda_A(p) \cdot [j = p_i] \right| = |a_{i, j} \cdot \det A_{i, j}| 再考虑正负号,即 τ ( p ) \tau(p) τ ( ξ ( p ) ) \tau(\xi(p)) 的奇偶性的差别,这取决于 p i p_i 前面大于 j j 的数量和 p i p_i 后面小于 j j 的数量,即 k = 1 i 1 [ p k > j ] + k = i + 1 n [ p k < j ] = k = 1 i 1 [ p k > j ] + ( j 1 k = 1 i 1 [ p k < j ] ) = k = 1 i 1 [ p k > j ] + j 1 ( i 1 k = 1 i 1 [ p k > j ] ) = 2 k = 1 i 1 [ p k > j ] + ( i + j ) i + j m o d    2 \begin{aligned} & \sum_{k = 1}^{i - 1} [p_k > j] + \sum_{k = i + 1}^{n}[p_k < j] \\ =& \sum_{k = 1}^{i - 1} [p_k > j] + \left( j - 1 - \sum_{k = 1}^{i - 1}[p_k < j] \right) \\ =& \sum_{k = 1}^{i - 1} [p_k > j] + j - 1 - \left( i - 1 - \sum_{k = 1}^{i - 1}[p_k > j] \right) \\ =& 2\sum_{k = 1}^{i - 1} [p_k > j] + (i + j) \equiv i + j \mod 2\end{aligned} 于是 τ ( p ) = ( 1 ) i + j τ ( ξ ( p ) ) \tau(p) = (-1)^{i + j} \tau(\xi(p)) ,则 p λ A ( p ) [ j = p i ] = ( 1 ) i + j a i , j det A i , j \sum_{p} \lambda_A(p) \cdot [j = p_i] = (-1)^{i + j} \cdot a_{i, j} \cdot \det A_{i, j}
于是 A = p λ A ( p ) = j = 1 n p λ A ( p ) [ j = p i ] = j = 1 n ( 1 ) i + j a i , j A i , j |A| = \sum_{p} \lambda_A(p) = \sum_{j = 1}^{n} \sum_{p} \lambda_A(p) \cdot [j = p_i] = \sum_{j = 1}^{n} (-1)^{i + j} a_{i, j} |A_{i, j}|

(百度百科“拉普拉斯展开”中有一个看不怎么懂的高端证明,但比较简洁,emmm 感觉上跟我这个自己 yy 的证明可能差不多)

线性性

可乘性

对于任意 1 i n 1 \leq i \leq n k k ,若 A = ( a 1 , 1 a 1 , 2 a 1 , n k a i , 1 k a i , 2 k a i , n a n , 1 a n , 2 a n , n ) A = \begin{pmatrix} a_{1, 1} & a_{1, 2} & \cdots & a_{1, n} \\ \vdots & \vdots & \ddots & \vdots \\ ka_{i, 1} & ka_{i, 2} & \cdots & ka_{i, n} \\ \vdots & \vdots & \ddots & \vdots \\ a_{n, 1} & a_{n, 2} & \cdots & a_{n, n} \end{pmatrix} B = ( a 1 , 1 a 1 , 2 a 1 , n a i , 1 a i , 2 a i , n a n , 1 a n , 2 a n , n ) B = \begin{pmatrix} a_{1, 1} & a_{1, 2} & \cdots & a_{1, n} \\ \vdots & \vdots & \ddots & \vdots \\ a_{i, 1} & a_{i, 2} & \cdots & a_{i, n} \\ \vdots & \vdots & \ddots & \vdots \\ a_{n, 1} & a_{n, 2} & \cdots & a_{n, n} \end{pmatrix} ,则 A = k B |A| = k|B|

证明:
根据定义,对于每个 p p λ A ( p ) = k λ B ( p ) \lambda_A(p) = k \cdot \lambda_B(p) ,得证。

可加性

对于任意 1 i n 1 \leq i \leq n k k a 1 , 1 a 1 , 2 a 1 , n a i 1 , 1 a i 1 , 2 a i 1 , n a i , 1 + b i , 1 a i , 2 + b i , 2 a i , n + b i , n a i + 1 , 1 a i + 1 , 2 a i + 1 , n a n , 1 a n , 2 a n , n = a 1 , 1 a 1 , 2 a 1 , n a i 1 , 1 a i 1 , 2 a i 1 , n a i , 1 a i , 2 a i , n a i + 1 , 1 a i + 1 , 2 a i + 1 , n a n , 1 a n , 2 a n , n + a 1 , 1 a 1 , 2 a 1 , n a i 1 , 1 a i 1 , 2 a i 1 , n b i , 1 b i , 2 b i , n a i + 1 , 1 a i + 1 , 2 a i + 1 , n a n , 1 a n , 2 a n , n \begin{vmatrix} a_{1, 1} & a_{1, 2} & \cdots & a_{1, n} \\ \vdots & \vdots & \ddots & \vdots \\ a_{i - 1, 1} & a_{i - 1, 2} & \cdots & a_{i - 1, n} \\ a_{i, 1} + b_{i, 1} & a_{i, 2} + b_{i, 2} & \cdots & a_{i, n} + b_{i, n} \\ a_{i + 1, 1} & a_{i + 1, 2} & \cdots & a_{i + 1, n} \\ \vdots & \vdots & \ddots & \vdots \\ a_{n, 1} & a_{n, 2} & \cdots & a_{n, n} \end{vmatrix} = \begin{vmatrix} a_{1, 1} & a_{1, 2} & \cdots & a_{1, n} \\ \vdots & \vdots & \ddots & \vdots \\ a_{i - 1, 1} & a_{i - 1, 2} & \cdots & a_{i - 1, n} \\ a_{i, 1} & a_{i, 2} & \cdots & a_{i, n} \\ a_{i + 1, 1} & a_{i + 1, 2} & \cdots & a_{i + 1, n} \\ \vdots & \vdots & \ddots & \vdots \\ a_{n, 1} & a_{n, 2} & \cdots & a_{n, n} \end{vmatrix} + \begin{vmatrix} a_{1, 1} & a_{1, 2} & \cdots & a_{1, n} \\ \vdots & \vdots & \ddots & \vdots \\ a_{i - 1, 1} & a_{i - 1, 2} & \cdots & a_{i - 1, n} \\ b_{i, 1} & b_{i, 2} & \cdots & b_{i, n} \\ a_{i + 1, 1} & a_{i + 1, 2} & \cdots & a_{i + 1, n} \\ \vdots & \vdots & \ddots & \vdots \\ a_{n, 1} & a_{n, 2} & \cdots & a_{n, n} \end{vmatrix}

证明:
对第 i i 行拉普拉斯展开即证。

不重性

对于 n × n ( n > 1 ) n \times n (n > 1) 的矩阵 A A ,若存在 1 i n , 1 j n , j j 1 \leq i \leq n, 1 \leq j \leq n, j \neq j 使得对于任意 1 k n 1 \leq k \leq n 都有 a i , k = a j , k a_{i, k} = a_{j, k} ,即矩阵中某两行对应相等,则 A = 0 |A| = 0

证明:
若某排列 p p 满足 p x = i , p y = j ( x < y ) p_x = i, p_y = j (x < y) ,记 q q p p 交换第 x , y x, y 个元素后得到的排列,即 q y = i , q x = j ( x < y ) q_y = i, q_x = j (x < y) ,那么 λ A ( p ) = λ A ( q ) \lambda_A(p) = -\lambda_A(q) ,因为两者逆序对个数相差 1 1
于是将所有 1 n 1 \sim n 的排列这样两两配对,即可使所有 λ A ( p ) \lambda_A(p) 抵消为 0 0 ,得证。

可倍加性

对于任意 1 i n , 1 j n , j j 1 \leq i \leq n, 1 \leq j \leq n, j \neq j
a 1 , 1 a 1 , 2 a 1 , n a i , 1 a i , 2 a i , n a n , 1 a n , 2 a n , n = a 1 , 1 a 1 , 2 a 1 , n a i , 1 + k a j , 1 a i , 2 + k a j , 2 a i , n + k a j , n a n , 1 a n , 2 a n , n \begin{vmatrix} a_{1, 1} & a_{1, 2} & \cdots & a_{1, n} \\ \vdots & \vdots & \ddots & \vdots \\ a_{i, 1} & a_{i, 2} & \cdots & a_{i, n} \\ \vdots & \vdots & \ddots & \vdots \\ a_{n, 1} & a_{n, 2} & \cdots & a_{n, n} \end{vmatrix} = \begin{vmatrix} a_{1, 1} & a_{1, 2} & \cdots & a_{1, n} \\ \vdots & \vdots & \ddots & \vdots \\ a_{i, 1} + ka_{j, 1} & a_{i, 2} + ka_{j, 2} & \cdots & a_{i, n} + ka_{j, n} \\ \vdots & \vdots & \ddots & \vdots \\ a_{n, 1} & a_{n, 2} & \cdots & a_{n, n} \end{vmatrix}

证明:
a 1 , 1 a 1 , 2 a 1 , n a i , 1 + k a j , 1 a i , 2 + k a j , 2 a i , n + k a j , n a n , 1 a n , 2 a n , n = a 1 , 1 a 1 , 2 a 1 , n a i , 1 a i , 2 a i , n a n , 1 a n , 2 a n , n + a 1 , 1 a 1 , 2 a 1 , n k a j , 1 k a j , 2 k a j , n a n , 1 a n , 2 a n , n ( ) = a 1 , 1 a 1 , 2 a 1 , n a i , 1 a i , 2 a i , n a n , 1 a n , 2 a n , n + k a 1 , 1 a 1 , 2 a 1 , n a j , 1 a j , 2 a j , n a n , 1 a n , 2 a n , n ( 线 ) = a 1 , 1 a 1 , 2 a 1 , n a i , 1 a i , 2 a i , n a n , 1 a n , 2 a n , n + 0 ( ) = a 1 , 1 a 1 , 2 a 1 , n a i , 1 a i , 2 a i , n a n , 1 a n , 2 a n , n \begin{aligned} & \begin{vmatrix} a_{1, 1} & a_{1, 2} & \cdots & a_{1, n} \\ \vdots & \vdots & \ddots & \vdots \\ a_{i, 1} + ka_{j, 1} & a_{i, 2} + ka_{j, 2} & \cdots & a_{i, n} + ka_{j, n} \\ \vdots & \vdots & \ddots & \vdots \\ a_{n, 1} & a_{n, 2} & \cdots & a_{n, n} \end{vmatrix} \\ =& \begin{vmatrix} a_{1, 1} & a_{1, 2} & \cdots & a_{1, n} \\ \vdots & \vdots & \ddots & \vdots \\ a_{i, 1} & a_{i, 2} & \cdots & a_{i, n} \\ \vdots & \vdots & \ddots & \vdots \\ a_{n, 1} & a_{n, 2} & \cdots & a_{n, n} \end{vmatrix} + \begin{vmatrix} a_{1, 1} & a_{1, 2} & \cdots & a_{1, n} \\ \vdots & \vdots & \ddots & \vdots \\ ka_{j, 1} & ka_{j, 2} & \cdots & ka_{j, n} \\ \vdots & \vdots & \ddots & \vdots \\ a_{n, 1} & a_{n, 2} & \cdots & a_{n, n} \end{vmatrix} & (可加性) \\ =& \begin{vmatrix} a_{1, 1} & a_{1, 2} & \cdots & a_{1, n} \\ \vdots & \vdots & \ddots & \vdots \\ a_{i, 1} & a_{i, 2} & \cdots & a_{i, n} \\ \vdots & \vdots & \ddots & \vdots \\ a_{n, 1} & a_{n, 2} & \cdots & a_{n, n} \end{vmatrix} + k\begin{vmatrix} a_{1, 1} & a_{1, 2} & \cdots & a_{1, n} \\ \vdots & \vdots & \ddots & \vdots \\ a_{j, 1} & a_{j, 2} & \cdots & a_{j, n} \\ \vdots & \vdots & \ddots & \vdots \\ a_{n, 1} & a_{n, 2} & \cdots & a_{n, n} \end{vmatrix} & (线性性) \\ =& \begin{vmatrix} a_{1, 1} & a_{1, 2} & \cdots & a_{1, n} \\ \vdots & \vdots & \ddots & \vdots \\ a_{i, 1} & a_{i, 2} & \cdots & a_{i, n} \\ \vdots & \vdots & \ddots & \vdots \\ a_{n, 1} & a_{n, 2} & \cdots & a_{n, n} \end{vmatrix} + 0 & (不重性) \\ =& \begin{vmatrix} a_{1, 1} & a_{1, 2} & \cdots & a_{1, n} \\ \vdots & \vdots & \ddots & \vdots \\ a_{i, 1} & a_{i, 2} & \cdots & a_{i, n} \\ \vdots & \vdots & \ddots & \vdots \\ a_{n, 1} & a_{n, 2} & \cdots & a_{n, n} \end{vmatrix} \end{aligned}

转置不变性

任意一个 n × n n \times n 的矩阵 A A ,满足 A T = A |A^T| = |A| A T A^T 表示矩阵 A A 的转置,即 a i , j a j , i a_{i, j} \to a_{j, i} (换句话说,将 A A 沿“左上 - 右下”对角线翻折即得到 A T A^T ),下同。

证明:
对于一个排列 p p ,根据转置的定义,令 ξ ( p ) = q   ( q p i = i ) \xi(p) = q\ (q_{p_i} = i) ,那么 λ A ( p ) = λ A T ( ξ ( p ) ) \lambda_A(p) = \lambda_{A^T}(\xi(p)) 。考虑 τ ( p ) \tau(p) τ ( ξ ( p ) ) \tau(\xi(p)) 的关系:
A i , p i   ( 1 i n ) A_{i, p_i}\ (1 \leq i \leq n) 记做特殊点,令 T p ( i ) \Tau_p(i) 表示 A i , p i A_{i, p_i} 的左下方的特殊点数量, T p ( i ) \Tau'_p(i) 表示 A i , p i A_{i, p_i} 的右上方的特殊点数量,则 τ ( p ) = i = 1 n T p ( i ) = i = 1 n T p ( i ) \tau(p) = \sum_{i = 1}^{n} \Tau_p(i) = \sum_{i = 1}^{n} \Tau'_p(i) ,由于是沿对角线翻折,所以显然 T p ( i ) = T q ( i ) \Tau_p(i) = \Tau'_q(i) (事实上只是把“左下方”的特殊点变到“右上方”,“右上方”变到“左下方”),于是 τ ( p ) = τ ( q ) \tau(p) = \tau(q)
因此 λ A ( p ) = λ A T ( ξ ( p ) ) \lambda_A(p) = \lambda_{A^T}(\xi(p)) ,得证。

可交换性

行可交换性

矩阵两行交换,矩阵的行列式值反号。

证明:
交换 A A 的两行 u , v u, v 可视为将 v v 行加到 u u 行上(①)得到一个新矩阵 A A' ,再从 A A' v v 行上减去 A A' u u 行(②),再将 A A' u u 行全部取反(③)。根据可倍加性,① 和 ② 均不改变行列式的值,根据可乘性,③ 操作使行列式的值反号,得证。

列可交换性

矩阵两列交换,矩阵行列式值反号。

证明:
根据转置不变性,先转置再用行可交换性交换两行,再转置回来即证。

优化行列式的计算

对矩阵 A A 的第一行进行拉普拉斯展开可以证明:当 A A 为一个上三角矩阵(即任意 i > j i > j ,满足 a i , j = 0 a_{i, j} = 0 )时: A = i = 1 n a i , i |A| = \prod_{i = 1}^{n} a_{i, i} 发现高斯消元用到的是可倍加性、可交换性,于是我们对 A A 高斯消元,可以得到一个上三角矩阵,这个矩阵的对角线乘积即为 A A 的行列式的绝对值(可交换性会使其反号)!于是我们做到了 O ( n 3 ) O(n^3) 求行列式的值。

矩阵树定理

前置定义

对于一个无向图 G = ( V , E ) G = (V, E) ,其中 V = n |V| = n (点数), E = m |E| = m (边数):

  • G G 的每条边任意分配一个方向,则它的关联矩阵(大小为 m × n m \times n B B b i , j = { 1   j     i   1   j     i   0 b_{i, j} = \begin{cases} 1 & 点\ j\ 是边\ i\ 的起点 \\ -1 & 点\ j\ 是边\ i\ 的终点 \\ 0 & 其他\end{cases}
  • G G 的基尔霍夫矩阵(大小为 n × n n \times n L L l i , j = { d i i = j e i , j i j l_{i, j} = \begin{cases} d_i & i = j \\ -e_{i, j} & i \neq j\end{cases} (其中 d i d_i 表示 i i 号点的度, e i , j e_{i, j} 连接点 i , j i, j 的边的数量)

一些引理

转置引理

L = B B T L = BB^T L , B , B T L, B, B^T 定义如上,乘法是矩阵乘法。

证明:
按照矩阵乘法的规则展开即证。
事实上如果定义两个序列相乘的结果是对应位置的乘积之和的话,矩阵乘法将 B B 的列两两相乘得到了一个新矩阵,这个矩阵就是 L L

连通性引理

引理 1

G G 是一个连通图,那么 L = 0 |L| = 0

证明:
G G 是连通图,那么根据定义 L L 的每一列的元素之和都为 0 0 ,再根据可倍加性,将其他所有行加到某一行上,这一行的元素就全部为零,再对这一行进行拉普拉斯展开,得到 L = 0 |L| = 0

引理 2

G G 不连通,则对任意 1 i n 1 \leq i \leq n L i , i = 0 |L_{i, i}| = 0

证明:
根据行列可交换性,我们将同一联通块的点换到一起,得到的新矩阵 L L' 满足 det L = det L |\det L'| = |\det L| ,并且 L L' 长这个样子: ( A 1 0 0 0 0 A 2 0 0 0 0 A 3 0 0 0 0 A k ) \begin{pmatrix} A_1 & \bold{0} & \bold{0} & \cdots & \bold{0} \\ \bold{0} & A_2 & \bold{0} & \cdots & \bold{0} \\ \bold{0} & \bold{0} & A_3 & \cdots & \bold{0} \\ \vdots & \vdots & \vdots & \ddots & \vdots \\ \bold{0} & \bold{0} & \bold{0} & \cdots & A_k\end{pmatrix} 0 \bold{0} 代表零矩阵, k k 是联通块的个数)
由于 G G 不连通,所以 k 2 k \geq 2 ,所以 L i , i L_{i, i} 至少包含一个 A x ( 1 x k ) A_x (1 \leq x \leq k) 。因为 A x A_x 是一个独立连通子图,所以 A x A_x 可以经过适当操作使得其对角线上某个元素为 0 0 ,进而 L i , i = 0 |L_{i, i}| = 0

引理 3

G G 是一个树,则对任意 1 i n 1 \leq i \leq n det L i , i = 1 |\det L_{i, i}| = 1

证明:
将这个树的结点拓扑排序,使得对于任意点 i i ,去除掉所有 1 i 1 1 \sim i - 1 的点后 i i 的度数为 1 1 ,这可以通过不断“摘掉”叶子结点实现。然后以这个顺序整理矩阵 L L 的各行得到 L L' ,我们对 L L' 模拟高斯消元的过程:
假设当前到了第 i i 行。没有消元时,由于我们拓扑排序了,那么 l i , i l'_{i, i} 右边(“右边”指所有 l i , j   ( j > i ) l'_{i, j}\ (j > i) ,“左边”“上面”“下面”同理)和下面有且仅有 1 1 1 -1 ,左边和上面有且仅有 l i , i 1 l'_{i, i} - 1 1 -1 ;由于我们用前 i 1 i - 1 行消掉了 l i , i l'_{i, i} 前面的 l i , i 1 l'_{i, i} - 1 1 -1 , 并且 l i , i l'_{i, i} 上面有 l i , i 1 l'_{i, i} - 1 1 -1 ,于是 l i , i l'_{i, i} 被加了 l i , i 1 l'_{i, i} - 1 1 -1 ,就变成了 1 1
按这样高斯消元下去,最后一个元素( l n , n l'_{n, n} ) 肯定是 0 0 。但我们要求的是 L i , i = L i , i |L_{i, i}| = |L'_{i, i}| ,它是 L L' 删去了一行一列,因此 L i , i L'_{i, i} 消元过后的最后一个( l n 1 , n 1 l''_{n - 1, n - 1} )是 1 1 。于是对角线上的元素全部是 1 1 ,得证。

Binet - Cauchy 定理

A A 是一个 n × m n \times m 的矩阵, B B 是一个 m × n m \times n 的矩阵,有 A B = { 0 n > m A B n = m 1 k 1 < k 2 < < k n m A ( 1 , 2 , , n ; k 1 , k 2 , , k n ) B ( k 1 , k 2 , , k n ; 1 , 2 , , n ) n < m |AB| = \begin{cases} 0 & n > m \\ |A| \cdot |B| & n = m\\ \sum\limits_{1 \leq k_1 < k_2 < \cdots < k_n \leq m} |A(1, 2, \cdots, n; k_1, k_2, \cdots, k_n)| \cdot |B(k_1, k_2, \cdots, k_n; 1, 2, \cdots, n)| & n < m \end{cases} A ( 1 , 2 , , n ; k 1 , k 2 , , k n ) A(1, 2, \cdots, n; k_1, k_2, \cdots, k_n) 表示由 A A 的第 k 1 , k 2 , , k n k_1, k_2, \cdots, k_n 列组成的矩阵; B ( k 1 , k 2 , , k n ; 1 , 2 , , n ) B(k_1, k_2, \cdots, k_n; 1, 2, \cdots, n) 表示由 B B 的第 k 1 , k 2 , , k n k_1, k_2, \cdots, k_n 行组成的矩阵)

证明:
我们构造两个辅助矩阵: M = ( A 0 I B ) ,    N = ( 0 A B I B ) M = \begin{pmatrix} A & \bold{0} \\ -I & B\end{pmatrix},\ \ N = \begin{pmatrix} \bold{0} & AB \\ -I & B\end{pmatrix} 其中 I I 是单位矩阵(“左上 - 右下”对角线为 1 1 的矩阵)可以发现 M = N |M| = |N| ,证明如下:
枚举 M M 矩阵的第 i   ( n + 1 i n + m ) i\ (n + 1 \leq i \leq n + m) 行,然后将第 i i 行乘以 a k 1 a_{k_1} 后加到第 1 1 行,乘以 a k 2 a_{k_2} 后加到第 2 2 行,……,乘以 a k n a_{k_n} 后加到第 n n 行。这样操作完后,根据矩阵乘法的定义, M M 变为了 N N ,又因为倍加不变性, M = N |M| = |N|
用定义式计算 N |N| ,由于只要 p p 选到 0 \bold{0} 中的元素 λ N ( p ) \lambda_N(p) 就为 0 0 ,所以只考虑 p 1 , p 2 , , p n [ m + 1 , m + n ] p_1, p_2, \cdots, p_n \in [m + 1, m + n] 的情况,又因为 A B AB 是个 n × n n \times n 的矩阵,所以 p n + 1 , p n + 2 , , p n + m [ 1 , m ] p_{n + 1}, p_{n + 2}, \cdots, p_{n + m} \in [1, m] ,所以 N = A B I = ( 1 ) m A B |N| = |AB| \cdot |-I| = (-1)^m |AB|
接下来分类计算 M |M| ,并证明该定理:

  • n > m n > m 时(图中 C = A B C = AB ,下同):
    n > m
    用定义式计算 M M ,发现排列 p p 无论如何取都会取到 0 \bold{0} 中的元素(原因就是 n > m n > m ),因此 M = 0 |M| = 0 ,即 ( 1 ) m A B = 0 (-1)^m |AB| = 0 ,即 A B = 0 |AB| = 0
  • n = m n = m 时:
    n = m
    用定义式计算 M M ,显然只有 p 1 , p 2 , , p n [ 1 , n ] p_1, p_2, \cdots, p_n \in [1, n] p n + 1 , p n + 2 , , p n + m [ n + 1 , n + m ] p_{n + 1}, p_{n + 2}, \cdots, p_{n + m} \in [n + 1, n + m] λ M ( p ) \lambda_M(p) 不为 0 0 ,于是 M = A B ( 1 ) n 2 |M| = |A| \cdot |B| \cdot (-1)^{n^2} 其中 ( 1 ) n 2 (-1)^{n^2} 即为分开计算 A , B A, B 将两者的排列合起来新形成的逆序对数。因而 A B ( 1 ) n 2 = A B ( 1 ) m |A| \cdot |B| \cdot (-1)^{n^2} = |AB| \cdot (-1)^m ,于是 A B = A B ( 1 ) m n 2 = A B ( 1 ) n n 2 = A B |A||B| = |AB|(-1)^{m - n^2} = |AB|(-1)^{n - n^2} = |AB|
  • n < m n < m 时:
    n < m
    这时候我们枚举 k 1 , k 2 , , k n [ 1 , m ] k_1, k_2, \cdots, k_n \in [1, m] (就是定理中的 k 1 , k 2 , , k n k_1, k_2, \cdots, k_n )表示在 A A 的各行中选的元素。为了使 λ M ( p ) 0 \lambda_M(p) \neq 0 I -I 中就只能选对角线上的元素,又因为 A A 中选的元素的正下方的那个 1 -1 肯定不能选(因为 p p 是排列),所以在 B B 被选了元素的行一定和 A A 被选了元素的列一样。举个例子,假设 n = 3 n = 3 ,红色是 A   /   B A\ /\ B 中含有被选中元素的列 / / 行。
    例子
    这样一来 A ( 1 , 2 , , n ; k 1 , k 2 , , k n ) |A(1, 2, \cdots, n;k_1, k_2, \cdots, k_n)| 的那些排列就和 B ( k 1 , k 2 , , k n ; 1 , 2 , , n ) |B(k_1, k_2, \cdots, k_n; 1, 2, \cdots, n)| 的那些排列形成了一一对应的函数关系,于是 M = 1 k 1 < k 2 < < k n m A ( 1 , 2 , , n ; k 1 , k 2 , , k n ) B ( k 1 , k 2 , , k n ; 1 , 2 , , n ) ( 1 ) m n + x |M| = \sum\limits_{1 \leq k_1 < k_2 < \cdots < k_n \leq m} |A(1, 2, \cdots, n; k_1, k_2, \cdots, k_n)| \cdot |B(k_1, k_2, \cdots, k_n; 1, 2, \cdots, n)| \cdot (-1)^{m - n + x} 其中 ( 1 ) m n (-1)^{m - n} I -I 的贡献, x x 是“新构成”的逆序对个数,所谓新构成,就是三部分的逆序对个数之和: I -I A A I -I B B A A B B 。由于枚举出的东西关于对角线对称,所以 I -I A A 之间的逆序对数等于 I -I B B 之间的逆序对数,于是 x x 可以转化为 A A B B 之间的逆序对数,即 n 2 n^2 。于是 M = 1 k 1 < k 2 < < k n m A ( 1 , 2 , , n ; k 1 , k 2 , , k n ) B ( k 1 , k 2 , , k n ; 1 , 2 , , n ) ( 1 ) m n + x = 1 k 1 < k 2 < < k n m A ( 1 , 2 , , n ; k 1 , k 2 , , k n ) B ( k 1 , k 2 , , k n ; 1 , 2 , , n ) ( 1 ) m n + n 2 = A B ( 1 ) m \begin{aligned} |M| &= \sum\limits_{1 \leq k_1 < k_2 < \cdots < k_n \leq m} |A(1, 2, \cdots, n; k_1, k_2, \cdots, k_n)| \cdot |B(k_1, k_2, \cdots, k_n; 1, 2, \cdots, n)| \cdot (-1)^{m - n + x} \\ &= \sum\limits_{1 \leq k_1 < k_2 < \cdots < k_n \leq m} |A(1, 2, \cdots, n; k_1, k_2, \cdots, k_n)| \cdot |B(k_1, k_2, \cdots, k_n; 1, 2, \cdots, n)| \cdot (-1)^{m - n + n^2} \\ &= |AB| (-1)^m \end{aligned} 于是 A B = 1 k 1 < k 2 < < k n m A ( 1 , 2 , , n ; k 1 , k 2 , , k n ) B ( k 1 , k 2 , , k n ; 1 , 2 , , n ) ( 1 ) n 2 n = 1 k 1 < k 2 < < k n m A ( 1 , 2 , , n ; k 1 , k 2 , , k n ) B ( k 1 , k 2 , , k n ; 1 , 2 , , n ) \begin{aligned} |AB| &= \sum\limits_{1 \leq k_1 < k_2 < \cdots < k_n \leq m} |A(1, 2, \cdots, n; k_1, k_2, \cdots, k_n)| \cdot |B(k_1, k_2, \cdots, k_n; 1, 2, \cdots, n)| \cdot (-1)^{n^2 - n} \\ &= \sum\limits_{1 \leq k_1 < k_2 < \cdots < k_n \leq m} |A(1, 2, \cdots, n; k_1, k_2, \cdots, k_n)| \cdot |B(k_1, k_2, \cdots, k_n; 1, 2, \cdots, n)| \end{aligned}

定理得证。

该证明参考了 Freopen 大佬的博客,详见参考资料。同样百度百科上有个很线代看不懂的证明。

矩阵树定理

对于 n n 个点 m m 条边的无向图 G G 以及任意一个 1 i n 1 \leq i \leq n L i i |L_{ii}| 即为它的生成树个数。

证明:
由转置引理 L i , i = B i , i B i , i T |L_{i, i}| = |B_{i, i} B_{i, i}^T| 因为 B i , i , B i , i T B_{i, i}, B_{i, i}^T 分别是大小为 ( n 1 ) × ( m 1 ) , ( m 1 ) × ( n 1 ) (n - 1) \times (m - 1), (m - 1) \times (n - 1) 的矩阵,所以由 Binet - Cauchy 定理得 L i , i = B i , i B i , i T = 1 k 1 < k 2 < < k n 1 m B i , i ( 1 , 2 , , n 1 ; k 1 , k 2 , , k n 1 ) B i , i T ( k 1 , k 2 , , k n 1 ; 1 , 2 , , n 1 ) |L_{i, i}| = |B_{i, i} B_{i, i}^T| = \sum_{1 \leq k_1 < k_2 < \cdots < k_{n - 1} \leq m} |B_{i, i}(1, 2, \cdots, n - 1; k_1, k_2, \cdots, k_{n - 1})| \cdot |B_{i, i}^T(k_1, k_2, \cdots, k_{n - 1}; 1, 2, \cdots, n - 1)| 由于 B i , i ( 1 , 2 , , n 1 ; k 1 , k 2 , , k n 1 ) = B i , i T ( k 1 , k 2 , , k n 1 ; 1 , 2 , , n 1 ) B_{i, i}(1, 2, \cdots, n - 1; k_1, k_2, \cdots, k_{n - 1}) = B_{i, i}^T(k_1, k_2, \cdots, k_{n - 1}; 1, 2, \cdots, n - 1) ,所以 L i , i = B i , i B i , i T = 1 k 1 < k 2 < < k n 1 m B i , i ( 1 , 2 , , n 1 ; k 1 , k 2 , , k n 1 ) B i , i T ( k 1 , k 2 , , k n 1 ; 1 , 2 , , n 1 ) = 1 k 1 < k 2 < < k n 1 m B i , i ( 1 , 2 , , n 1 ; k 1 , k 2 , , k n 1 ) 2 \begin{aligned} |L_{i, i}| = |B_{i, i} B_{i, i}^T| &= \sum_{1 \leq k_1 < k_2 < \cdots < k_{n - 1} \leq m} |B_{i, i}(1, 2, \cdots, n - 1; k_1, k_2, \cdots, k_{n - 1})| \cdot |B_{i, i}^T(k_1, k_2, \cdots, k_{n - 1}; 1, 2, \cdots, n - 1)| \\ &= \sum_{1 \leq k_1 < k_2 < \cdots < k_{n - 1} \leq m} |B_{i, i}(1, 2, \cdots, n - 1; k_1, k_2, \cdots, k_{n - 1})|^2 \end{aligned} 由关联矩阵的定义可知, B = B i , i ( 1 , 2 , , n 1 ; k 1 , k 2 , , k n 1 ) B' = B_{i, i}(1, 2, \cdots, n - 1; k_1, k_2, \cdots, k_{n - 1}) 就是由图 G G 的边 k 1 , k 2 , , k n 1 k_1, k_2, \cdots, k_{n - 1} 构成的子图 G G' 的关联矩阵。由连通性引理二得,当 G G' 不连通时, B = 0 |B'| = 0 ;由连通性引理三得,当 G G' 是树时, det B = 1 |\det B'| = 1 ,因此 B i , i ( 1 , 2 , , n 1 ; k 1 , k 2 , , k n 1 ) 2 = 1 |B_{i, i}(1, 2, \cdots, n - 1; k_1, k_2, \cdots, k_{n - 1})|^2 = 1 当且仅当由图 G G 的边 k 1 , k 2 , , k n 1 k_1, k_2, \cdots, k_{n - 1} 构成的子图是树,得证。

参考资料

Matrix - Tree 定理(生成树计数)的另类证明和简单拓展 - MoebiusMeow
矩阵树定理 - Freopen
Matrix - Tree 矩阵树定理 - Lucky_Glass
拉普拉斯展开 - 百度百科
Binet - Cauchy 定理 - 百度百科

猜你喜欢

转载自blog.csdn.net/C20190102/article/details/107084966
今日推荐